The answer I got for this problem was 10.8 (multiplied 18 and 60 then divided by 100) however, it stated that my answer is incorrect. Did I not round correctly or is my entire answer wrong?

The Answer I Got For This Problem Was 10.8 (multiplied 18 And 60 Then Divided By 100) However, It Stated

Answers

Answer 1

9514 1404 393

Answer:

  30 mL

Step-by-step explanation:

The meaning of 60% solution is ...

  alcohol/(total volume) = 0.60

You are told the amount of alcohol is 18 mL, so you have ...

  (18 mL)/(total volume) = 0.60

Multiplying by total volume gives ...

  18 mL = 0.60 × (total volume)

Then dividing by 0.60, you have ...

  (18 mL)/0.60 = total volume = 30 mL

_____

Additional comment

You need to apply what you know about the real world to consider whether your answer makes any sense. If alcohol is a little more than half of the volume, and the amount of alcohol is 18 mL, then the total volume cannot be less than 18 mL. In fact, you expect it to be not quite double 18 mL.

An answer of 10.8 mL makes no sense in this context. You cannot have less total solution than you have of something that makes up a part of the total solution.


Related Questions

Which diagram represents the hypothesis of the converse of corresponding angles theorem?

Answers

Answer:

the first diagram

Step-by-step explanation:

first one

In a bag of mini candy bars, 40% of the candy bars are Snickers. There are 120 Snickers in the bag. How many total mini candy bars are in the bag?​

Answers

Answer:

The amswer is 48.

Step-by-step explanation:

120×0.40=48

How much is 12,856 ounces in pounds ?

Answers

Answer : 803.5 pounds

There are 16 ounces in a pound, so just divide 12,856 by 16, giving you 803.5.

A group of 3 boys is sharing 4 small pans of rice krispie treats. What is the total amount of rice krispie treats each boy will get?

Answers

Answer:

each boy will get 1.33333333333 of a pan

Step-by-step explanation:

Answer:

Each boy in that group would get about 1 and 1 third of a pan of treats or 1.33 repeating

Step-by-step explanation:

To find this you would take the numbers of pans (4) and divide it by the number of boys (3).

4 / 3 = 1.33333

Which number's estimate written as a single digit times a power of 10 will have a negative exponent?

Answers

105 i hope this helps if not then i’m sorry

Heyy!! Can someone help me please!!

3 (5x + 2) - 2 (4x -4)

I don’t know what to doooo!!

Answers

Answer:

7x + 14

Step-by-step explanation:

the first thing to do is expand the parentheses/brackets.

3(5x + 2) -2(4x - 4) will be

3(5x) + 3(2) -2(4x) -2(-4)

= 15x + 6 - 8x + 8

collect like terms

15x - 8x + 6 + 8 = 7x + 14

the answer is 7x + 14

Answer:

3(5x+2)-2(4x-4)

15x+6-8x+8

15x-8x+6+8

7x+14

Simplify:......................................................

Answers

Answer:

...

Step-by-step explanation:...

The cut off number is 2x-1

Please please help me please I really need help please just tap on picture and you will see the question

Answers

Answer:

No , it is not a right angle triangle

Step-by-step explanation:

according to the pythagoras theorem in right angled triangle sum of square of two sides is equal to the square of it's hypotenuse.

using pythagoras theorem

a^2 + b^2 = c^2

9^2 + 16^2 = 25^2

81 + 256 = 625

337 = 625

since sum of square of two smallest sides of a triangle is not equal to the square of it's hypotenuse it can be concluded that the given figure does not form right angle triangle.

Find the area of the following shape:

Answers

Answer:

36cm^2

Step-by-step explanation:

total area: 6x(4+3)=42

total area excluding the space: 42-(2x3)=36

Answer:

36 cm squared

Step-by-step explanation:

To solve this problem, I first construct a line. (shown in yellow in the first photo)

I then find the area of the top rectangle. (6 cm * 4 cm = 24 cm squared.)

Next, I find the area of the lower rectangle. But...to do that I have to find the length of the line that I constructed. To do this, I do  6cm-2cm=4cm.

Then I can find the area of the lower rectangle. (4cm*3cm=12cm squared.)

add up the area of both of the rectangles and.........12+24=36 cm squared

Two workers paint lines for angled parking spaces

Answers

Answer:

2nd option

Step-by-step explanation:

The lines are parallel because the angles are congruent since they are Alternate exterior angles.

Given points (-3;-6), G(3; -2) and H(6; 1); determine:
(a) The equation of line FG

Answers

Answer:

The equation of line FG is [tex]y = \frac{2}{3}x - 4[/tex]

Step-by-step explanation:

Equation of a line:

The equation of a line has the following format:

[tex]y = mx + b[/tex]

In which m is the slope and b is the y-intercept.

F(-3;-6), G(3; -2)

When we have two points, the slope is given by the change in y divided by the change in x. So

Change in y : -2 - (-6) = -2 + 6 = 4

Change in x: 3 - (-3) = 3 + 3 = 6

Slope: [tex]m = \frac{4}{6} = \frac{2}{3}[/tex]

So

[tex]y = \frac{2}{3}x + b[/tex]

Finding b:

(3; -2) means that when [tex]x = 3, y = -2[/tex]. We use this to find b.

[tex]y = \frac{2}{3}x + b[/tex]

[tex]-2 = \frac{2}{3}(3) + b[/tex]

[tex]2 + b = -2[/tex]

[tex]b = -4[/tex]

The equation of line FG is [tex]y = \frac{2}{3}x - 4[/tex]

A geneticist conducts an experiment with peas, one sample of offspring consisted of 450 green peas and 157 yellow peas. Based on these results, estimate the probability of getting an offspring pea that is green.

Answers

Answer: 0.738

Step-by-step explanation:

11. Mendelian Genetics. When Mendel conducted his famous genetics experiments with peas, one sample of offspring consisted of 428 green peas and 152 yellow peas. Based on those results, estimate the probability of getting an offspring pea that is green. Is the result reasonably close to the value that was expected?

p0 = 428/(428 + 152) = 0.737931

If you round your answer of 0.737931 to three decimals you will

get 0.738.

Let V be the set of all 3x3 matrices with Real number entries, with the usual definitions of scalar multiplication and vector addition. Consider whether V is a vector space over C. Mark all true statements (there may be more than one).

a. The scalar closure axiom is satisfied
b. The additive inverse axiom is not satisfied
c The additive inverse axiom is satisfied
d. The additive closure axiom is not satisfied
e. The scalar closure axiom is not satisfied
f. The additive closure axiom is satsified
g. V is not a vector space over C
h. V is a vector space over C
i. The zero axiom is satisfied
j. The zero axiom is not satisfied

Answers

Answer:

the Scalar Closure axiom is not satisfiedV is not a Vector Space of CThe Additive Closure axiom is satisfied.

Step-by-step explanation:

According to the Question,

Given That, Let V be the set of all 3x3 matrices with Real number entries, with the usual definitions of scalar multiplication and vector addition. Consider whether V is a vector space over C.For V is a vector space over C and V is Set of 3x3 Matrices with Real entries.

Then, For any u,w ∈ V ⇒ u+w ∈ V

And u∈V and z∈C ⇒ z u ∈ V

So, If we take any z= i ∈ C

and V be any 3x3 matrices with Real entrices.

then, z,v ∉ V  ∴z,v Has Complex entries

So, the Scalar Closure axiom is not satisfied

Hence, V is not a Vector Space of C

Any u,w ∈ W ⇒ u+w ∈ V

So, The Additive Closure axiom is satisfied.

Joe drives for 3 hours and covers 201 miles. In miles per hour, how fast was he driving?​

Answers

Answer:

67 mph

Step-by-step explanation:

201/3 = 67

Cierto individuo, al fallecer, repartió sus ahorros del siguiente modo; la mitad a
su esposa; el tercio a su único hijo; la décima parte a un sobrino, y dos millones
de nuevos soles a una beneficencia ¿cuánto poseía?

Answers

Answer:

El individuo poseía treinta millones.

Step-by-step explanation:                              

Podemos expresar la repartición del individuo como sigue:

[tex] x = \frac{x}{2} + \frac{x}{3} + \frac{x}{10} + 2000000 [/tex]

Dado que la suma de la repartición de la mitad que le da a su esposa (x/2), del tercio que le da a su único hijo (x/3), de la décima parte que le da a un sobrino (x/10) y de los dos millones que le da a una beneficiencia debe ser igual al monto incial que tenía (x).

Resolviendo la ecuación de arriba para "x" tenemos:

[tex] x - \frac{x}{2} - \frac{x}{3} - \frac{x}{10} = 2000000 [/tex]

[tex] x = 30000000 [/tex]

Por lo tanto, el individuo poseía treinta millones.

Espero que te sea de utilidad!                  

If f(x) = x -2 and g(x) = 2x – 6, then g(4)/f(3) =​

Answers

Answer:

Step-by-step explanation:

(2×4-6)/(3-2)=2

Answer:

[tex]{ \tt{f(x) = x - 2}} \\ { \bf{f(3) = 3 - 2 = 1}} \\ \\ { \tt{g(x) = 2x - 6}} \\ { \bf{g(4) = 2(4) - 6 = 2}} \\ \\ { \boxed{ \tt{ \frac{g(4)}{f(3)} = \frac{2}{1} = 2}}}[/tex]

N/A questions does not exist

Answers

Answer:

ok

Step-by-step explanation:

why did you put it

The diagram shows a right-angled triangle.
xo
26 cm
17 cm
Find the size of angle x.
Give your answer correct to 1 decimal place.

Answers

Answer:

Diagram? I don't see a diagram.

Where is the diagram?

Step-by-step explanation:

Câu 1 (2 điểm). Cho hệ các vector

U = {(1,2,2); (0,-1,-1);(2,3,3);(1,0,0).

a) tìm số chiều và một cơ sở W của không gian con sinh bởi hệ vector U

b) tìm tham số k để u=(2,3,k^2 +1) là một tổ hợp tuyến tính cảu W, và suy ra [u]w

giải hộ mình với

Answers

Answer:

Step-by-step explanation:

11x+7y=17
solve for y

Answers

[tex]\implies {\blue {\boxed {\boxed {\purple {\sf {\: y = \frac{17 - 11x}{7} }}}}}}[/tex]

[tex]\large\mathfrak{{\pmb{\underline{\red{Step-by-step\:explanation}}{\red{:}}}}}[/tex]

[tex]\\11x + 7y = 17[/tex]

[tex] \\➺ \: 7y = 17 - 11x[/tex]

[tex]\\➺ \: y = \frac{17 - 11x}{7} [/tex]

[tex]\bold{ \green{ \star{ \orange{Mystique35}}}}⋆[/tex]

Find (x,y) such that (8,2)(x,y) = (28,22)

Answers

Answer:

(x,y) = (3.5, 11)

Step-by-step explanation:

(8,2)(x,y) = (28,22)

Dot product, so:

[tex](8,2)(x,y) = (8x,2y) = (28,22)[/tex]

Then

[tex]8x = 28[/tex]

[tex]x = \frac{28}{8} = 3.5[/tex]

And

[tex]2y = 22[/tex]

[tex]y = \frac{22}{2} = 11[/tex]

So

(x,y) = (3.5, 11)

Which rules of exponents will be used to evallate this expression? Select three options.

Answers

I need the answer choices and the expression you’re referring to

What is the area of this figure?

Answers

Answer:

90km² only if it is parallelogram

Step-by-step explanation:

base = 9km

height=10km

area of parallelogram = b x h

=9km x 10km

=90km²

Answer:

A = 90km2

Step-by-step explanation:

Area of a rhombus is:

1. A = s x h (if given side and height)

2. A = 1/2 a x b (if given lengths of diagonals)

3. A = s^2 sin A (if given side and length)

Therefore from your problem, height and side is given thus, you'll use number 1

A = s x h

A = 9km x 10km = 90km2

Help me with this answer I don’t it

Answers

Answer:

f(-2) = g(-2) this is the answer

 Marsha has a bag that contains 4 green marbles, 8 yellow marbles , and 20 red marbles . If she chooses one marble from the bag, what is the probability that the marble is not red?

PLEASE HELP IF YOURE GOOD AT GEOMETRY!!

Answers

Answer:

C. 3/8

HOPE THIS HELPS :)

Answer:

c. 3/8

Step-by-step explanation:

first you need the denomerator by adding all marbles together which equals 32. now for the munerator you need the sum of the green and yellow marbles. this equals 12. so your fraction is 12/32. next we simplify. we can divide both numbers by 4. getting us a fraction of 3/8.

Consider a Poisson distribution with μ = 6.

a. Write the appropriate Poisson probability function.
b. Compute f (2).
c. Compute f (1).
d. Compute P(x≥2)

Answers

Answer:

a) [tex]P(X = x) = \frac{e^{-6}*6^{x}}{(x)!}[/tex]

b) f(2) = 0.04462

c) f(1) = 0.01487

d) [tex]P(X \geq 2) = 0.93803[/tex]

Step-by-step explanation:

In a Poisson distribution, the probability that X represents the number of successes of a random variable is given by the following formula:

[tex]P(X = x) = \frac{e^{-\mu}*\mu^{x}}{(x)!}[/tex]

In which

x is the number of successes

e = 2.71828 is the Euler number

[tex]\mu[/tex] is the mean in the given interval.

In this question:

[tex]\mu = 6[/tex]

a. Write the appropriate Poisson probability function.

Considering [tex]\mu = 6[/tex]

[tex]P(X = x) = \frac{e^{-6}*6^{x}}{(x)!}[/tex]

b. Compute f (2).

This is P(X = 2). So

[tex]P(X = x) = \frac{e^{-6}*6^{x}}{(x)!}[/tex]

[tex]P(X = 2) = \frac{e^{-6}*6^{2}}{(2)!} = 0.04462[/tex]

So f(2) = 0.04462

c. Compute f (1).

This is P(X = 1). So

[tex]P(X = x) = \frac{e^{-6}*6^{x}}{(x)!}[/tex]

[tex]P(X = 1) = \frac{e^{-6}*6^{1}}{(1)!} = 0.01487[/tex]

So f(1) = 0.01487.

d. Compute P(x≥2)

This is:

[tex]P(X \geq 2) = 1 - P(X < 2)[/tex]

In which:

[tex]P(X < 2) = P(X = 0) + P(X = 1) + P(X = 2)[/tex]

[tex]P(X = x) = \frac{e^{-6}*6^{x}}{(x)!}[/tex]

[tex]P(X = 0) = \frac{e^{-6}*6^{0}}{(0)!} = 0.00248[/tex]

[tex]P(X = 1) = \frac{e^{-6}*6^{1}}{(1)!} = 0.01487[/tex]

[tex]P(X = 2) = \frac{e^{-6}*6^{2}}{(2)!} = 0.04462[/tex]

Then

[tex]P(X < 2) = P(X = 0) + P(X = 1) + P(X = 2) = 0.00248 + 0.01487 + 0.04462 = 0.06197[/tex]

[tex]P(X \geq 2) = 1 - P(X < 2) = 1 - 0.06197 = 0.93803[/tex]

So

[tex]P(X \geq 2) = 0.93803[/tex]

What is the range of f(x) = 3X?
O A. All real numbers greater than 3
O B. All real numbers greater than or equal to 3
O C. All positive real numbers
O D. All real numbers

Answers

D all real numbers (-∞, ∞)

Complete the table of ordered pairs for the linear y=2x-8

Answers

Given:

Consider the below figure attached with this question.

The linear equation is:

[tex]y=2x-8[/tex]

To find:

The values to complete the table of ordered pairs for the given linear equation.

Solution:

We have,

[tex]y=2x-8[/tex]

Substituting [tex]x=0[/tex] in the given equation, we get

[tex]y=2(0)-8[/tex]

[tex]y=0-8[/tex]

[tex]y=-8[/tex]

So, the value for first blank is -8.

Substituting [tex]y=-2[/tex] in the given equation, we get

[tex]-2=2x-8[/tex]

[tex]-2+8=2x[/tex]

[tex]\dfrac{6}{2}=x[/tex]

[tex]3=x[/tex]

So, the value for second blank is 3.

Substituting [tex]x=2[/tex] in the given equation, we get

[tex]y=2(2)-8[/tex]

[tex]y=4-8[/tex]

[tex]y=-4[/tex]

So, the value for third blank is -4.

Therefore, the required complete table is:

     x           y

     0         -8

    3         -2

     2          -4

Suppose that g(x)= f(x)+ 6. Which statement best compares the graph of g(x) with the graph of f(x)?

A. The graph of g(x) is the graph of f(x) shifted 6 units down.

B. The graph of g(x) is the graph of f(x) shifted to the right.

C. The graph of g(x) is the graph of f(x) shifted 6 units to the left.

D. The graph of g(x) is the graph of f(x) shifted 6 units up.

Answers

Answer:

D

Step-by-step explanation:

The + 6 moves it up 6 units.

The correct answer is (D) "The graph of g(x) is the graph of f(x) shifted 6 units up."

What is the function?

A relationship between a group of inputs and one output is referred to as a function. In plain English, a function is an association between inputs in which each input is connected to precisely one output. A domain, codomain, or range exists for every function. Typically, f(x), where x is the input, is used to represent a function.

When we add a constant to a function, such as in the case of g(x) = f(x) + 6, it will shift the graph of f(x) upward by 6 units.

This is because, for any value of x, the value of f(x) will be added to 6, resulting in a vertical shift of the entire graph.

Option (A) is incorrect because adding 6 to f(x) would shift the graph up, not down.

Option (B) is incorrect because adding a constant to a function does not cause it to shift horizontally.

Option (C) is incorrect because adding 6 to f(x) would shift the graph right, not left.

D. The graph of g(x) is the graph of f(x) shifted 6 units up. Adding a constant term to a function will shift the graph of the function vertically. In this case, adding 6 to f(x) will shift the graph of f(x) upward by 6 units, resulting in the graph of g(x).

Learn more about function here:

https://brainly.com/question/29633660

#SPJ7

HELP HELP HELP
Solve this

Answers

Answer:

What is the cos theta for, i would use sin to solve for theta and then we would get 41.25 degrees.

Step-by-step explanation:

Other Questions
write an equation for the sentence: the sum of six and twice a number is equal to sixteen minus the number. URGENT! ONLY HAVE 5 MINUTES! help me find the size of the rhombus :) GHI Corporation, a California corporation, has a six-person board. At a regular board meeting, only two directors attend. No notice was sent to any of the directors. The two attending call directors Alice and Bob and put them on a conference call. The four talk about the corporation buying Blackacre and then all agree to a resolution for GHI to buy Blackacre from Third Party. The Bylaws of GHI state that an action of the board requires the consent of a majority of the directors present at a meeting, and that a quorum is a majority of the authorized directors.Select one:a. the purchase is authorized because a quorum was present and a majority of those present approved the action.b. the purchase is not authorized, since all real estate transactions require shareholder approvalc. the purchase is not authorized because prior written notice must be sent to each directord. the purchase is not authorized because a quorum was not present at the board meetinge. Two of the above are correct. El verbo tiene dos partes, la terminacin y la raiz. Cul verbo NO est divido correctamente ? A. comie -ron B. camin -asteC. trat- aronY porque.Gracias This is my introduction are you to help me Most of us, at at least one point of our life, have experienced a time in a moving vehicle where someone almost got into an accident or did get into one. Neither fun, but here is something you did not know. This year alone there have been 6,357 fatal car accidents, even during the Pandemic, where we should all be staying home. Today we will be talking about the effect automobiles have on our society today including, deadly accidents, too fast of transportation, and toxic fumes damaging the ozone layer. The attraction forces between atoms or molecules of a liquid become ______ during turbulent flow. Which right determine who is responsible for managing the resources? Blank rights determine who is responsible for managing the resources What is this? Ianjansjshshsjssjsjjss 1.how can you categorize the buyers who are not price sensitive ? WILL MARK YOU IF YOU ANSWER SO PLEASE HELP Please select the word from the list that best fits the definitionDaniel es muy aventurero. Nada le da miedo. The sentiments expressed in the proclamation would have been most widely condemned by White residents of coastal South Carolina Answer A: coastal South Carolina A northern California Answer B: northern California B western New York Answer C: western New York C western Virginia If a normal distribution has a mean of 154 and a standard deviation of 15,what is the value that has a z-score of 1.6? What three parts do both the U.S. Constitution and the Florida Constitution include?A)Beginning, Middle, and EndB)Preamble, Amendments, and RevisionsPreamble, Articles, and AmendmentsD)Introduction, Structures, and Changes The table shows the height of water in a pool as it is being filled. A table showing Height of Water in a Pool with two columns and six rows. The first column, Time in minutes, has the entries, 2, 4, 6, 8, 10. The second column, Height in inches, has the entries, 8, 12, 16, 20, 24. The slope of the line through the points is 2. Which statement describes how the slope relates to the height of the water in the pool? The height of the water increases 2 inches per minute. The height of the water decreases 2 inches per minute. The height of the water was 2 inches before any water was added. The height of the water will be 2 inches when the pool is filled. Question 7. His parents object strongly his playing too much computer games. A. at B. to C. in D. againstQuestion 8. It gets when the winter is coming.A. cold and cold B. cold and colder C. more and more cold D. colder and colderQuestion 9. She has just bought .A. a Swiss beautiful brand-new watch B. a Swiss brand-new beautiful watch C. a brand-new beautiful Swiss watch D. a beautiful brand-new Swiss watchQuestion 10: When I last saw him, he________in London. A. is living B. has been living C. was living D. lived Select the correct answer.Alan's dogs have a total of 24 legs (l). If each dog has 4 legs, which equation gives the number of dogs (d) that Alan has?A. 4 d = 24B. 4 l = 24C. 4 + d = 24D. 4 + l = 24does anyone know the answer to that question i need it ASAP !!!! What Is Soil Tunneling? Question 9 of 25Who owns the factories under the system of capitalism?A. WorkersB. Private companiesC. ConsumersD. The governmentCOM When was this created?